Đến nội dung

Hình ảnh

$\frac{a+b+c}{3}-\sqrt[3]{abc}\leq max {(\sqrt{a}-\sqrt{b})^{2}}$

bất đẳng thức

  • Please log in to reply
Chủ đề này có 2 trả lời

#1
Hahahahahahahaha

Hahahahahahahaha

    Trung sĩ

  • Thành viên
  • 104 Bài viết

cho a,b,c là các số thực dương. CMR:
$\frac{a+b+c}{3}-\sqrt[3]{abc}\leq max {(\sqrt{a}-\sqrt{b})^{2},(\sqrt{b}-\sqrt{c})^{2},(\sqrt{c}-\sqrt{a})^{2}}$

                                                                                    (đề chọn đội tuyển dự thi Toán quốc tế, Mỹ năm 2000)


Bài viết đã được chỉnh sửa nội dung bởi Hahahahahahahaha: 02-12-2023 - 21:58

       Nơi nào có ý chí, nơi đó có con đường. Nếu trong triệu khả năng, có một khả năng bạn làm được điều gì đó, bất cứ điều gì, để giữ thứ bạn muốn không kết thúc, hãy làm đi. Hãy cạy cửa mở, hoặc thậm chí nếu cần, hãy nhét chân vào cửa để giữ cửa mở.

        Where there is a will, there is a way. If there is a chance in a million that you can do something, anything, to keep what you want from ending, do it. Pry the door open or, if need be, wedge your foot in that door and keep it open.

                                                                                                                                                             Pauline Kael

 

 


#2
hanguyen445

hanguyen445

    Thượng sĩ

  • Thành viên
  • 241 Bài viết

cho a,b,c là các số thực dương. CMR:
$\frac{a+b+c}{3}-\sqrt[3]{abc}\leq max {(\sqrt{a}-\sqrt{b})^{2},(\sqrt{b}-\sqrt{c})^{2},(\sqrt{c}-\sqrt{a})^{2}}$

                                                                                    (đề chọn đội tuyển dự thi Toán quốc tế, Mỹ năm 2000)

Do bài toán đối xứng với ba biến $a, b, c$, không giảm tổng quát giả sử $a\ge b\ge c$.

Lúc này ta suy ra được $(\sqrt{a}-\sqrt{c})^2=\text{Max}( (\sqrt{a}-\sqrt{c})^2;(\sqrt{a}-\sqrt{b})^2;(\sqrt{b}-\sqrt{c})^2)$

Do đó BĐT cần chứng minh tương đương

 

$$\frac{a+b+c}{3}-\sqrt[3]{abc}\le (\sqrt{c}-\sqrt{a})^{2}$$

 

Đặt $(x;y;z)=(\sqrt[6]{a},\sqrt[6]{b},\sqrt[6]{c})$ và hiển nhiên $x\ge y\ge z\ge 0$,  khi đó bất đẳng thức viết lại thành 

$$x^6+y^6+z^6-3(xyz)^2\le 3(x^3-z^3)^2$$

Ta có:

 

$x^6+y^6+z^6-3(xyz)^2=(x^2+y^2+z^2)[(x^2-z^2)^2+(y^2-x^2)(y^2-z^2)$

 

$3(x^3-z^3)^2=3(x-z)^2(x^2+xz+z^2)^2$

 

Hiển nhiên $(y^2-x^2)(y^2-z^2)\le 0$ với mọi $x\ge y\ge z$. Như vậy bất đẳng thức đúng nếu ta chứng minh được

$$(x^2+y^2+z^2)(x+z)^2\le 3(x^2+xz+z^2)^2(**)$$

Ta có $(x^2+xz+z^2)\ge\frac{3}{4} (x+z)^2$. Do đó ta suy ra được $VP(**)\ge\frac{9}{4}(x+z)^2(x^2+xz+z^2)$, nên BĐT đúng nếu

$$9(x^2+xz+z^2)\ge 4(x^2+y^2+z^2) (3*)$$

Hiển nhiên: $9x^2>8x^2>4(x^2+y^2)$ và $9(xz+z^2)\ge 4z^2$. Cộng vế ta suy ra được (3*) được chứng minh.

 

Vậy phép chứng minh hoàn tất.


Bài viết đã được chỉnh sửa nội dung bởi hanguyen445: 02-12-2023 - 23:45


#3
nhungvienkimcuong

nhungvienkimcuong

    Thiếu úy

  • Hiệp sỹ
  • 678 Bài viết

cho a,b,c là các số thực dương. CMR:
$\frac{a+b+c}{3}-\sqrt[3]{abc}\leq max {(\sqrt{a}-\sqrt{b})^{2},(\sqrt{b}-\sqrt{c})^{2},(\sqrt{c}-\sqrt{a})^{2}}$

                                                                                    (đề chọn đội tuyển dự thi Toán quốc tế, Mỹ năm 2000)

Bài này khá lỏng, dễ thấy 

\[\max \left( (\sqrt{a}-\sqrt{b})^{2},(\sqrt{b}-\sqrt{c})^{2},(\sqrt{c}-\sqrt{a})^{2}\right )\ge \frac{\sum (\sqrt{a}-\sqrt{b})^{2}}{3}=\frac{2}{3}\left(\sum a-\sum \sqrt{ab}\right).\]

Như vậy ta cần chứng minh

\[\frac{1}{3}\sum a-\sqrt[3]{abc}\le\frac{2}{3}\left(\sum a-\sum \sqrt{ab}\right)\iff \sum a+3\sqrt[3]{abc}\ge 2\sum\sqrt{ab}.\]

Tuy nhiên theo bất đẳng thức Schur thì

\[\sum a+3\sqrt[3]{abc}\ge \sum\sqrt[3]{ab}(\sqrt[3]{a}+\sqrt[3]{b})\ge \sum\sqrt[3]{ab}\cdot 2\sqrt{\sqrt[3]{ab}}=2\sum \sqrt{ab}.\]


Đừng khóc vì chuyện đã kết thúc hãy cười vì chuyện đã xảy ra ~O) 
Thật kì lạ anh không thể nhớ đến tên mình mà chỉ nhớ đến tên em :wub:
Chúa tạo ra vũ trụ của con người còn em tạo ra vũ trụ của anh :ukliam2:






Được gắn nhãn với một hoặc nhiều trong số những từ khóa sau: bất đẳng thức

1 người đang xem chủ đề

0 thành viên, 1 khách, 0 thành viên ẩn danh